LSAT and Law School Admissions Forum

Get expert LSAT preparation and law school admissions advice from PowerScore Test Preparation.

 Administrator
PowerScore Staff
  • PowerScore Staff
  • Posts: 8916
  • Joined: Feb 02, 2011
|
#25634
Complete Question Explanation
(The complete setup for this game can be found here: lsat/viewtopic.php?t=7800)

The correct answer choice is (D)

Prephrasing the answer to this Global, Must Be True question is challenging, in part because the
game lacks any “breakthrough” inferences to target in our prephrase. Without a prephrase,
proceed by the process of elimination: any answer choice that is not necessarily true will be
incorrect. Remember to reuse your prior work: while the setups to your previous questions cannot
be used to prove what must be true, they can show what is not necessarily true. Use them, but
with caution!

Answer choice (A): This answer choice may seem attractive, because G is visited either first or
second in all of the local diagrams created so far. However, this answer choice is incorrect. Even
if G is not visited first or second, the following solution would not conflict with any of the rules
in the game:
june07_game_3_#17_diagram_1.png
Answer choice (B): This answer choice is disproven by the second local setup to Question #13,
where M was neither second nor third:
june07_game_3_#17_diagram_2.png
The remaining three answer choices test your understanding of the two Numerical Distributions
discussed earlier:
june07_game_3_#17_diagram_3.png
Answer choice (C): This answer choice is incorrect, because G can also be visited three times, as
shown in the 3-2-1-1 distribution. The possibility of visiting G three times is also suggested by
the second setup to Question #13.

Answer choice (D): This is the correct answer choice. If J were visited more than twice, then G
would have to be visited more than twice, because each visit to J requires a prior visit to G (fourth
rule). However, two destinations cannot be visited three times each. Therefore, there can be no
more than two visits to J.

Answer choice (E): This answer choice is not necessarily true, because T can be visited three
times in the 3-2-1-1 distribution. The hypothetical solution disproving answer choice (A) here can
also be used to disprove answer choice (E).
You do not have the required permissions to view the files attached to this post.
 saranash1
  • Posts: 168
  • Joined: May 21, 2013
|
#9655
section 1
17.For the question i picked answer choice B because of my set up for number 14 where I have M in either 2 or 3.

set up looks like this

G T M G J M T
or
G M T G J M T

Is b not the answer because it is not a must that M go in 2 or 3? It just can go in two or three?
Last edited by saranash1 on Thu Jun 06, 2013 2:19 am, edited 1 time in total.
 Ron Gore
PowerScore Staff
  • PowerScore Staff
  • Posts: 220
  • Joined: May 15, 2013
|
#9692
Thanks for your question, saranash1.

The best way to think about this question is numbers, rather than the specific placement of a variable.

We often use the concept of numerical distributions to help us identify the numerical limitations of a game. We would describe this game as Unbalanced, and it is Underfunded. Specifically, there are four destinations and seven weeks. That difference requires that some of the destinations will be reused. Because we know each destination must be visited at least once, and that M has to be visited exactly twice, our base distribution is 2:1:1:1. This means that after the minimum number of visits to each destination is met, there are two destinations left to assign.

Because the fourth rule tells us J :arrow: GJ, we know we cannot have J selected more than twice. If we select J just once, we must have G at least once, which is fine. If we have J twice, we must have G at least twice, which still works, because it takes up the two spare destinations we have after meeting the base distribution. However, if we have J three times, we must have G at least three times, and there just aren't that many weeks available.

Generally speaking, the numerical distribution in this game is more limiting than the placement of specific destinations in certain weeks, and for that reason it would be helpful to focus on answer choices (C), (D) and (E), which involve numbers, rather than on (A) and (B), which involve placement of specific destinations in certain weeks.

As you suggest in your question, (B) is incorrect because it is not a requirement that M go in either week 2 or week 3. However, please be clear with what (B) says. For (B) to be incorrect, it must be possible to have a valid template in which M is in neither 2 nor 3.

Hope this helps,

Ron
 saranash1
  • Posts: 168
  • Joined: May 21, 2013
|
#9720
Thanks that makes sense! How was this game set up and the rules diagramed?
 Steve Stein
PowerScore Staff
  • PowerScore Staff
  • Posts: 1153
  • Joined: Apr 11, 2011
|
#9729
Hi saranash1,

Thanks for your response. As you probably know, this is a basic linear game; it would be helpful for you to provide your diagram, and we can confirm its accuracy and make any necessary suggestions.

Thanks!

~Steve
 saranash1
  • Posts: 168
  • Joined: May 21, 2013
|
#9761
g <____J
M g M ( didn't know how to show that it goes somewhere in between M but doesn't have to go immediately after and before m
M :arrow: 2x

GJMT 4

_ _ _ M/t/g _ G/J/m T
no J No T
1 2 3 4 5 6 7
 Steve Stein
PowerScore Staff
  • PowerScore Staff
  • Posts: 1153
  • Joined: Apr 11, 2011
|
#9765
Hey saranash1,

Thanks for your response. That looks good to me, although I would diagram the rule about Jamaica as a conditional:

Any voyage to J will be immediately preceded by G:

J :arrow: GJ

In other words, if the boat goes to J, it will be as part of the GJ block.

I hope that's helpful--please let me know whether this is clear--thanks!

~Steve
 saranash1
  • Posts: 168
  • Joined: May 21, 2013
|
#9768
Yes that helps!

does M G M show that it goes somewhere in between M but doesn't have to go immediately after and before m because it doesn't have a block around it?
 Steve Stein
PowerScore Staff
  • PowerScore Staff
  • Posts: 1153
  • Joined: Apr 11, 2011
|
#9776
Yes, that's right--as long as you understand what's relayed, I think the way you diagrammed that one was good.

I hope that's helpful!

~Steve
 saranash1
  • Posts: 168
  • Joined: May 21, 2013
|
#9778
thanks!

Get the most out of your LSAT Prep Plus subscription.

Analyze and track your performance with our Testing and Analytics Package.